If $a$ is of order $3$ mod a prime $p$, then …











up vote
1
down vote

favorite












The question says:




Prove that if $a$ is of order $3$ modulo a prime $p$, then $1+a+a^2equiv 0 pmod p$. Moreover, $a+1$ is of order $6$.




For the First Part:



The typical idea is to start with $a^3 equiv 1 pmod p to a^3 -1 equiv 0 pmod p$. Factoring the term on the left hand side, the rest is straightforward.



However, I need to check the following idea:




$$1+a+a^2 equiv a^3+a^2+a equiv a(1+a+a^2)equiv a^2(1+a+a^2)$$
$$equiv a^3(1+a+a^2) equiv 0 pmod p$$



Since, by the hypothesis, $a^3$ cannot be zero modulo the prime $p$, the desired result holds.




If this idea holds true, it can be generalized to the following result:




if $a$ is of order $k$, then, modulo prime $p$, then $1+a+a^2+cdots + a^{k-1}$ is divisible by $p$.




Is it??



For the Second Part:



I can see that:



$$1+a+a^2 equiv 0 to 1+a equiv -a^2 pmod p$$



However, does this implies anything? Given that $a$ is of order $3$, but what about $-a$??



Please Help, and Thanks in advance,,










share|cite|improve this question
























  • math.stackexchange.com/questions/220493/…
    – lab bhattacharjee
    Nov 23 at 7:16










  • @labbhattacharjee, I would like to see your comments about thoughts on the first part.
    – Maged Saeed
    Nov 23 at 8:12















up vote
1
down vote

favorite












The question says:




Prove that if $a$ is of order $3$ modulo a prime $p$, then $1+a+a^2equiv 0 pmod p$. Moreover, $a+1$ is of order $6$.




For the First Part:



The typical idea is to start with $a^3 equiv 1 pmod p to a^3 -1 equiv 0 pmod p$. Factoring the term on the left hand side, the rest is straightforward.



However, I need to check the following idea:




$$1+a+a^2 equiv a^3+a^2+a equiv a(1+a+a^2)equiv a^2(1+a+a^2)$$
$$equiv a^3(1+a+a^2) equiv 0 pmod p$$



Since, by the hypothesis, $a^3$ cannot be zero modulo the prime $p$, the desired result holds.




If this idea holds true, it can be generalized to the following result:




if $a$ is of order $k$, then, modulo prime $p$, then $1+a+a^2+cdots + a^{k-1}$ is divisible by $p$.




Is it??



For the Second Part:



I can see that:



$$1+a+a^2 equiv 0 to 1+a equiv -a^2 pmod p$$



However, does this implies anything? Given that $a$ is of order $3$, but what about $-a$??



Please Help, and Thanks in advance,,










share|cite|improve this question
























  • math.stackexchange.com/questions/220493/…
    – lab bhattacharjee
    Nov 23 at 7:16










  • @labbhattacharjee, I would like to see your comments about thoughts on the first part.
    – Maged Saeed
    Nov 23 at 8:12













up vote
1
down vote

favorite









up vote
1
down vote

favorite











The question says:




Prove that if $a$ is of order $3$ modulo a prime $p$, then $1+a+a^2equiv 0 pmod p$. Moreover, $a+1$ is of order $6$.




For the First Part:



The typical idea is to start with $a^3 equiv 1 pmod p to a^3 -1 equiv 0 pmod p$. Factoring the term on the left hand side, the rest is straightforward.



However, I need to check the following idea:




$$1+a+a^2 equiv a^3+a^2+a equiv a(1+a+a^2)equiv a^2(1+a+a^2)$$
$$equiv a^3(1+a+a^2) equiv 0 pmod p$$



Since, by the hypothesis, $a^3$ cannot be zero modulo the prime $p$, the desired result holds.




If this idea holds true, it can be generalized to the following result:




if $a$ is of order $k$, then, modulo prime $p$, then $1+a+a^2+cdots + a^{k-1}$ is divisible by $p$.




Is it??



For the Second Part:



I can see that:



$$1+a+a^2 equiv 0 to 1+a equiv -a^2 pmod p$$



However, does this implies anything? Given that $a$ is of order $3$, but what about $-a$??



Please Help, and Thanks in advance,,










share|cite|improve this question















The question says:




Prove that if $a$ is of order $3$ modulo a prime $p$, then $1+a+a^2equiv 0 pmod p$. Moreover, $a+1$ is of order $6$.




For the First Part:



The typical idea is to start with $a^3 equiv 1 pmod p to a^3 -1 equiv 0 pmod p$. Factoring the term on the left hand side, the rest is straightforward.



However, I need to check the following idea:




$$1+a+a^2 equiv a^3+a^2+a equiv a(1+a+a^2)equiv a^2(1+a+a^2)$$
$$equiv a^3(1+a+a^2) equiv 0 pmod p$$



Since, by the hypothesis, $a^3$ cannot be zero modulo the prime $p$, the desired result holds.




If this idea holds true, it can be generalized to the following result:




if $a$ is of order $k$, then, modulo prime $p$, then $1+a+a^2+cdots + a^{k-1}$ is divisible by $p$.




Is it??



For the Second Part:



I can see that:



$$1+a+a^2 equiv 0 to 1+a equiv -a^2 pmod p$$



However, does this implies anything? Given that $a$ is of order $3$, but what about $-a$??



Please Help, and Thanks in advance,,







number-theory elementary-number-theory modular-arithmetic divisibility primitive-roots






share|cite|improve this question















share|cite|improve this question













share|cite|improve this question




share|cite|improve this question








edited Nov 23 at 4:20

























asked Nov 23 at 4:16









Maged Saeed

762316




762316












  • math.stackexchange.com/questions/220493/…
    – lab bhattacharjee
    Nov 23 at 7:16










  • @labbhattacharjee, I would like to see your comments about thoughts on the first part.
    – Maged Saeed
    Nov 23 at 8:12


















  • math.stackexchange.com/questions/220493/…
    – lab bhattacharjee
    Nov 23 at 7:16










  • @labbhattacharjee, I would like to see your comments about thoughts on the first part.
    – Maged Saeed
    Nov 23 at 8:12
















math.stackexchange.com/questions/220493/…
– lab bhattacharjee
Nov 23 at 7:16




math.stackexchange.com/questions/220493/…
– lab bhattacharjee
Nov 23 at 7:16












@labbhattacharjee, I would like to see your comments about thoughts on the first part.
– Maged Saeed
Nov 23 at 8:12




@labbhattacharjee, I would like to see your comments about thoughts on the first part.
– Maged Saeed
Nov 23 at 8:12










1 Answer
1






active

oldest

votes

















up vote
2
down vote



accepted










For the second part, taking from the first part that $1+a = -a^2pmod pimplies (1+a)^6= (-a^2)^6 = a^{12}pmod p= (a^3)^4 =1^4pmod p=1pmod p$ . Thus $1+a$ is of order $6$ as claimed. And if $a$ is of order $3$ then $(-a)^3 = -a^3 = -1 = p-1pmod p$. And from this we have: $(-a)^6 = ((-a)^3)^2 = (-1)^2 = 1pmod p$. So $-a$ is of order $6$ as well.






share|cite|improve this answer























  • Thanks, but how to be sure that $6$ is the least power congruent to $1 pmod p$ for $(1+a)$??
    – Maged Saeed
    Nov 23 at 4:28












  • I need also to make sure my thoughts are correct for the first part. Thanks in advance.
    – Maged Saeed
    Nov 23 at 4:29






  • 1




    @MagedSaeed: You can prove it yourself that if $k$ is such that $(a+1)^k = 1pmod p implies k = 6$. In fact, such $k$ must satisfies $k mid 6 implies k = 1,2,3$. And none of these can satisfy since $a$ is of order $3$.
    – DeepSea
    Nov 23 at 4:35












  • Oh, thanks,, I see.
    – Maged Saeed
    Nov 23 at 4:37











Your Answer





StackExchange.ifUsing("editor", function () {
return StackExchange.using("mathjaxEditing", function () {
StackExchange.MarkdownEditor.creationCallbacks.add(function (editor, postfix) {
StackExchange.mathjaxEditing.prepareWmdForMathJax(editor, postfix, [["$", "$"], ["\\(","\\)"]]);
});
});
}, "mathjax-editing");

StackExchange.ready(function() {
var channelOptions = {
tags: "".split(" "),
id: "69"
};
initTagRenderer("".split(" "), "".split(" "), channelOptions);

StackExchange.using("externalEditor", function() {
// Have to fire editor after snippets, if snippets enabled
if (StackExchange.settings.snippets.snippetsEnabled) {
StackExchange.using("snippets", function() {
createEditor();
});
}
else {
createEditor();
}
});

function createEditor() {
StackExchange.prepareEditor({
heartbeatType: 'answer',
convertImagesToLinks: true,
noModals: true,
showLowRepImageUploadWarning: true,
reputationToPostImages: 10,
bindNavPrevention: true,
postfix: "",
imageUploader: {
brandingHtml: "Powered by u003ca class="icon-imgur-white" href="https://imgur.com/"u003eu003c/au003e",
contentPolicyHtml: "User contributions licensed under u003ca href="https://creativecommons.org/licenses/by-sa/3.0/"u003ecc by-sa 3.0 with attribution requiredu003c/au003e u003ca href="https://stackoverflow.com/legal/content-policy"u003e(content policy)u003c/au003e",
allowUrls: true
},
noCode: true, onDemand: true,
discardSelector: ".discard-answer"
,immediatelyShowMarkdownHelp:true
});


}
});














draft saved

draft discarded


















StackExchange.ready(
function () {
StackExchange.openid.initPostLogin('.new-post-login', 'https%3a%2f%2fmath.stackexchange.com%2fquestions%2f3009968%2fif-a-is-of-order-3-mod-a-prime-p-then%23new-answer', 'question_page');
}
);

Post as a guest















Required, but never shown

























1 Answer
1






active

oldest

votes








1 Answer
1






active

oldest

votes









active

oldest

votes






active

oldest

votes








up vote
2
down vote



accepted










For the second part, taking from the first part that $1+a = -a^2pmod pimplies (1+a)^6= (-a^2)^6 = a^{12}pmod p= (a^3)^4 =1^4pmod p=1pmod p$ . Thus $1+a$ is of order $6$ as claimed. And if $a$ is of order $3$ then $(-a)^3 = -a^3 = -1 = p-1pmod p$. And from this we have: $(-a)^6 = ((-a)^3)^2 = (-1)^2 = 1pmod p$. So $-a$ is of order $6$ as well.






share|cite|improve this answer























  • Thanks, but how to be sure that $6$ is the least power congruent to $1 pmod p$ for $(1+a)$??
    – Maged Saeed
    Nov 23 at 4:28












  • I need also to make sure my thoughts are correct for the first part. Thanks in advance.
    – Maged Saeed
    Nov 23 at 4:29






  • 1




    @MagedSaeed: You can prove it yourself that if $k$ is such that $(a+1)^k = 1pmod p implies k = 6$. In fact, such $k$ must satisfies $k mid 6 implies k = 1,2,3$. And none of these can satisfy since $a$ is of order $3$.
    – DeepSea
    Nov 23 at 4:35












  • Oh, thanks,, I see.
    – Maged Saeed
    Nov 23 at 4:37















up vote
2
down vote



accepted










For the second part, taking from the first part that $1+a = -a^2pmod pimplies (1+a)^6= (-a^2)^6 = a^{12}pmod p= (a^3)^4 =1^4pmod p=1pmod p$ . Thus $1+a$ is of order $6$ as claimed. And if $a$ is of order $3$ then $(-a)^3 = -a^3 = -1 = p-1pmod p$. And from this we have: $(-a)^6 = ((-a)^3)^2 = (-1)^2 = 1pmod p$. So $-a$ is of order $6$ as well.






share|cite|improve this answer























  • Thanks, but how to be sure that $6$ is the least power congruent to $1 pmod p$ for $(1+a)$??
    – Maged Saeed
    Nov 23 at 4:28












  • I need also to make sure my thoughts are correct for the first part. Thanks in advance.
    – Maged Saeed
    Nov 23 at 4:29






  • 1




    @MagedSaeed: You can prove it yourself that if $k$ is such that $(a+1)^k = 1pmod p implies k = 6$. In fact, such $k$ must satisfies $k mid 6 implies k = 1,2,3$. And none of these can satisfy since $a$ is of order $3$.
    – DeepSea
    Nov 23 at 4:35












  • Oh, thanks,, I see.
    – Maged Saeed
    Nov 23 at 4:37













up vote
2
down vote



accepted







up vote
2
down vote



accepted






For the second part, taking from the first part that $1+a = -a^2pmod pimplies (1+a)^6= (-a^2)^6 = a^{12}pmod p= (a^3)^4 =1^4pmod p=1pmod p$ . Thus $1+a$ is of order $6$ as claimed. And if $a$ is of order $3$ then $(-a)^3 = -a^3 = -1 = p-1pmod p$. And from this we have: $(-a)^6 = ((-a)^3)^2 = (-1)^2 = 1pmod p$. So $-a$ is of order $6$ as well.






share|cite|improve this answer














For the second part, taking from the first part that $1+a = -a^2pmod pimplies (1+a)^6= (-a^2)^6 = a^{12}pmod p= (a^3)^4 =1^4pmod p=1pmod p$ . Thus $1+a$ is of order $6$ as claimed. And if $a$ is of order $3$ then $(-a)^3 = -a^3 = -1 = p-1pmod p$. And from this we have: $(-a)^6 = ((-a)^3)^2 = (-1)^2 = 1pmod p$. So $-a$ is of order $6$ as well.







share|cite|improve this answer














share|cite|improve this answer



share|cite|improve this answer








edited Nov 23 at 4:42

























answered Nov 23 at 4:25









DeepSea

70.8k54487




70.8k54487












  • Thanks, but how to be sure that $6$ is the least power congruent to $1 pmod p$ for $(1+a)$??
    – Maged Saeed
    Nov 23 at 4:28












  • I need also to make sure my thoughts are correct for the first part. Thanks in advance.
    – Maged Saeed
    Nov 23 at 4:29






  • 1




    @MagedSaeed: You can prove it yourself that if $k$ is such that $(a+1)^k = 1pmod p implies k = 6$. In fact, such $k$ must satisfies $k mid 6 implies k = 1,2,3$. And none of these can satisfy since $a$ is of order $3$.
    – DeepSea
    Nov 23 at 4:35












  • Oh, thanks,, I see.
    – Maged Saeed
    Nov 23 at 4:37


















  • Thanks, but how to be sure that $6$ is the least power congruent to $1 pmod p$ for $(1+a)$??
    – Maged Saeed
    Nov 23 at 4:28












  • I need also to make sure my thoughts are correct for the first part. Thanks in advance.
    – Maged Saeed
    Nov 23 at 4:29






  • 1




    @MagedSaeed: You can prove it yourself that if $k$ is such that $(a+1)^k = 1pmod p implies k = 6$. In fact, such $k$ must satisfies $k mid 6 implies k = 1,2,3$. And none of these can satisfy since $a$ is of order $3$.
    – DeepSea
    Nov 23 at 4:35












  • Oh, thanks,, I see.
    – Maged Saeed
    Nov 23 at 4:37
















Thanks, but how to be sure that $6$ is the least power congruent to $1 pmod p$ for $(1+a)$??
– Maged Saeed
Nov 23 at 4:28






Thanks, but how to be sure that $6$ is the least power congruent to $1 pmod p$ for $(1+a)$??
– Maged Saeed
Nov 23 at 4:28














I need also to make sure my thoughts are correct for the first part. Thanks in advance.
– Maged Saeed
Nov 23 at 4:29




I need also to make sure my thoughts are correct for the first part. Thanks in advance.
– Maged Saeed
Nov 23 at 4:29




1




1




@MagedSaeed: You can prove it yourself that if $k$ is such that $(a+1)^k = 1pmod p implies k = 6$. In fact, such $k$ must satisfies $k mid 6 implies k = 1,2,3$. And none of these can satisfy since $a$ is of order $3$.
– DeepSea
Nov 23 at 4:35






@MagedSaeed: You can prove it yourself that if $k$ is such that $(a+1)^k = 1pmod p implies k = 6$. In fact, such $k$ must satisfies $k mid 6 implies k = 1,2,3$. And none of these can satisfy since $a$ is of order $3$.
– DeepSea
Nov 23 at 4:35














Oh, thanks,, I see.
– Maged Saeed
Nov 23 at 4:37




Oh, thanks,, I see.
– Maged Saeed
Nov 23 at 4:37


















draft saved

draft discarded




















































Thanks for contributing an answer to Mathematics Stack Exchange!


  • Please be sure to answer the question. Provide details and share your research!

But avoid



  • Asking for help, clarification, or responding to other answers.

  • Making statements based on opinion; back them up with references or personal experience.


Use MathJax to format equations. MathJax reference.


To learn more, see our tips on writing great answers.





Some of your past answers have not been well-received, and you're in danger of being blocked from answering.


Please pay close attention to the following guidance:


  • Please be sure to answer the question. Provide details and share your research!

But avoid



  • Asking for help, clarification, or responding to other answers.

  • Making statements based on opinion; back them up with references or personal experience.


To learn more, see our tips on writing great answers.




draft saved


draft discarded














StackExchange.ready(
function () {
StackExchange.openid.initPostLogin('.new-post-login', 'https%3a%2f%2fmath.stackexchange.com%2fquestions%2f3009968%2fif-a-is-of-order-3-mod-a-prime-p-then%23new-answer', 'question_page');
}
);

Post as a guest















Required, but never shown





















































Required, but never shown














Required, but never shown












Required, but never shown







Required, but never shown

































Required, but never shown














Required, but never shown












Required, but never shown







Required, but never shown







Popular posts from this blog

How do I know what Microsoft account the skydrive app is syncing to?

When does type information flow backwards in C++?

Grease: Live!